Tổng quan về chuyên đề đa thức

  1. Tác giả: LTTK CTV
    Đánh giá: ✪ ✪ ✪ ✪ ✪

    TỔNG QUAN CHUYÊN ĐỀ ĐA THỨC

    PHẦN I: ĐA THỨC
    Đa thức là một trong những khái niệm trung tâm của toán học. Trong chương trình phổ thông, chúng ta đã làm quen với khái niệm đa thức từ bậc trung học cơ sở, từ những phép cộng, trừ, nhân đa thức đến phân tích đa thức ra thừa số, dùng sơ đồ Horner để chia đa thức, giải các phương trình đại số.

    Bài giảng này sẽ hệ thống hoá lại những kiến thức cơ bản nhất về đa thức 1 biến, các dạng toán thường gặp về đa thức. Ở cuối bài sẽ đề cập 1 cách sơ lược nhất về đa thức nhiều biến.


    1. Đa thức và các phép toán trên đa thức

    1.1. Định nghĩa. Đa thức trên trường số thực là biểu thức có dạng

    $P(x)=a_{n}x^{n}+a_{n-1}x^{n-1}+...+a_{1}x+a_{0},$ trong đó $a_{i}\in \mathbb{R}$ và $a_{n}\neq 0.$
    $a_{1}$ được gọi là các hệ số của đa thức, trong đó $a_{n}$ được gọi là hệ số cao nhất và $a_{0}$ được gọi là hệ số tự do.
    $n$ được gọi là bậc của đa thức và kí hiệu là $n=deg(P).$ Ta quy ước bậc của đa thức hằng $P(x)=a_{0}$ với mọi $x$ là bằng $0$ nếu $a_{0}\neq 0$ và bằng nếu $a_{0}=0.$

    Để tiện lợi cho việc viết các công thức, ta quy ước với đa thức $P(x)$ bậc $n$ thì vẫn có các hệ số $a_{k}$ với $k>n,$ nhưng chúng đều bằng $0.$

    Tập hợp các đa thức $1$ biến trên trường các số thực được kí hiệu là $R[x].$ Nếu các hệ số được lấy trên tập hợp các số hữu tỷ, các số nguyên thì ta có khái niệm đa thức với hệ số hữu tỷ, đa thức với hệ số nguyên và tương ứng là các tập hợp $Q[x],Z[x].$

    1.2. Đa thức bằng nhau

    Hai đa thức $P(x)=\sum_{k=0}^{m}a_{k}x^{k},Q(x)=\sum_{k=0}^{n}b_{k}x^{k}$ bằng nhau khi và chỉ khi
    $m=n$ và $a_{k}=b_{k}\forall k=0,1,2,...,m.$

    1.3. Phép cộng trừ đa thức.

    Cho hai đa thức $P(x)=\sum_{k=0}^{m}a_{k}x^{k},Q(x)=\sum_{k=0}^{n}b_{k}x^{k}$. Khi đó phép cộng và trừ hai đa thức $P(x)$ và $Q(x)$ được thực hiện theo từng hệ số của $x_{k},$ tức là
    $P(x)\pm Q(x)=\sum_{k=0}^{max\begin{Bmatrix} m,n \end{Bmatrix}}(a_{k}\pm b_{k})x^{k}$
    Ví dụ: $(x^{3}+3x^{2}-x+2)+(x^{2}+x-1)=x^{3}+4x^{2}+1$

    1.4. Phép nhân đa thức

    Cho hai đa thức $P(x)=\sum_{k=0}^{m}a_{k}x^{k},Q(x)=\sum_{k=0}^{n}b_{k}x^{k}$. Khi đó $P(x).Q(x)$ là một đa thức có bậc $m+n$ và có các hệ số xác định bởi
    $c_{k}=\sum_{i=0}^{k}a_{i}b_{k-i}$​
    Ví dụ:
    $(x^{3}+x^{2}+3x+2)(x^{2}+3x+1)=(1.1)x^{5}+(1.3+1.1)x^{4}+(1.1+1.3+3.1)x^{3}+(1.1+3.3+2.1)x^{2}+(3.1+2.3)x+(2.1)=x^{5}+4x^{4}+7x^{3}+12x^{2}+9x+1$

    1.5. Bậc của tổng, hiệu và tích của các đa thức

    Từ các định nghĩa trên đây, dễ dàng suy ra các tính chất sau đây

    Đinh lý 1. Cho $P(x),Q(x)$ là các đa thức bậc $m,n$ tương ứng. Khi đó
    a) $deg(P\pm Q)\leq max\begin{Bmatrix} m,n \end{Bmatrix}$ trong đó nếu $deg(P)\neq deg(Q)$ thì dấu bằng xảy ra. Trong trường hợp $m=n$ thì $deg(P\pm Q)$ có thể nhận bất cứ giá trị nào $\leq m$.
    b) $deg(P.Q)=m+n$

    1.6. Phép chia có dư

    Định lý 2. Với hai đa thức $P(x)$ và $Q(x)$ bất kỳ, trong đó $deg(Q)\geq 1,$ tồn tại duy nhất các đa thức $S(x)$ và $R(x)$ thoả mãn đồng thời các điều kiện:
    $i)$ $P(x)=Q(x).S(x)+R(x)$
    $ii)$ $deg(R)<deg(Q)$
    Chứng minh. Tồn tại. Ta có thể chứng minh bằng quy nạp theo $m=deg(P).$ Nếu $deg(P)<deg(Q)$ thì ta có thể chọn $S(x)\equiv 0)$ và $R(x)=P(x)$ thoả mãn đồng thời các điều kiện $i)$ và $ii)$. Giả sử $m\geq n$ và định lý đã được chứng minh với đa thức có bậc nhỏ hơn $m.$ Ta chứng minh định lý đúng với các đa thức bậc $m.$ Giả sử
    $P(x)=\sum_{k=0}^{m}a_{k}x^{k},Q(x)=\sum_{k=0}^{n}b_{k}x^{k}$​
    Xét đa thức
    $H(x)=P(x)-\dfrac{a_{m}}{b_{n}}x^{m-n}Q(x)$
    $=(a_{m}x^{m}+a_{m-1}x^{m-1}+...+a_{1}x+a_{0})-\dfrac{a_{m}}{b_{n}}x^{m-n}(b_{n}x^{n}+...+b_{0})$
    $=\begin{pmatrix} a_{m-1}-\dfrac{a_{m}b_{n-1}}{b_{n}} \end{pmatrix}x^{m-1}+...$​
    Do hệ số của $x^{m}$ ở hai đa thức bị triệt tiêu nên bậc của $H(x)$ không vượt quá $m-1$.
    Theo giả thiết quy nạp, tồn tại các đa thức $S*(x)$ và $R*(x)$ sao cho
    $H(x)=S*(x).Q(x)+R*(x)$​
    Nhưng khi đó
    $P(x)=H(x)+\frac{a_{m}}{b_{n}}x^{m-n}Q(x)=(\frac{a_{m}}{b_{n}}x^{m-n}+S*(x))+R*(x)$​
    Vậy đặt $S(x)=\frac{a_{m}}{b_{n}}x^{m-n}+S*(x)$ và $R(x)=R*(x)$ ta được biểu diễn cần tìm cho $P(x)$
    Duy nhất. Giả sử ta có hai biểu diễn $P(x)=S(x).Q(x)+R(x)$ và $P(x)=S*(x).Q(x)+R*(x)$ thoả mãn điều kiện ii). Khi đó
    $Q(x).(S(x)-S*(x))=R*(x)-R(x).$​
    Ta có, theo điều kiện $ii)$ và định lý $1$ thì $deg(R*(x)-R(x))<deg(Q).$ Mặt khác, nếu $S(x)-S*(x)$ không đồng nhất bằng $0$ thì
    $deg(Q(x).(S(x)-S*(x)))=deg(Q(x))+deg(S(x)-S*(x))\geq deg(Q).$​
    Mâu thuẫn vì hai vế bằng nhau.

    Theo ký hiệu của định lý thì $S(x)$ được gọi là thương số và $R(x)$ được gọi là dư số trong phép chia $P(x)$ cho $Q(x).$

    Phép chứng minh nói trên cũng cho chúng ta thuật toán tìm thương số và dư số của phép chia hai đa thức, gọi là phép chia dài (long division) hay sơ đồ Horner

    Ví dụ: Thực hiện phép chia $3x^{3}-2x^{2}+4x+7$ cho $x^{2}+2x$ được $3x-8$, dư $20x+7.$

    1,7. Sự chia hết. Ước và bội.

    Trong phép chia $P(x)$ cho $Q(x),$ nếu dư số $R(x)$ đồng nhất bằng $0$ thì ta nói rằng đa thức $P(x)$ chia hết cho đa thức $Q(x).$ Như vậy, $P(x)$ chia hết cho $Q(x)$ nếu tồn tại đa thức $S(x)$ sao cho $P(x) = Q(x).S(x).$ Trong trường hợp này ta cũng nói $Q(x)$ chia hết $P(x),$ $Q(x)$ là ước của $P(x)$ hoặc $P(x)$ là bội của $Q(x).$ Ký hiệu tương ứng là $Q(x) | P(x)$ và $P(x)\vdots Q(x).$

    Cho $P(x)$ và $Q(x)$ là các đa thức khác $0.$ Ước chung lớn nhất của $P(x)$ và $Q(x)$ là đa thức $D(x)$ thoả mãn đồng thời các điều kiện sau:
    $i)$ $D(x)$ là đa thức đơn khởi, tức là có hệ số cao nhất bằng $1$
    $ii)$ $D(x)$ là ước chung của $P(x)$ và $Q(x),$ tức là $D(x) | P(x)$ và $D(x) | Q(x)$
    $iii)$ Nếu $D’(x)$ cũng là ước chung của $P(x)$ và $Q(x)$ thì $D(x)$ cũng là ước của $D’(x).$

    Tương tự, ta có khái niệm bội chung nhỏ nhất của hai đa thức.
    Cho $P(x)$ và $Q(x)$ là các đa thức khác $0.$ Bội chung lớn nhất của $P(x)$ và $Q(x)$ là đa thức $M(x)$ thoả mãn đồng thời các điều kiện sau:
    $iv)$ $M(x)$ là đa thức đơn khởi, tức là có hệ số cao nhất bằng $1$
    $v)$ $M(x)$ là bội chung của $P(x)$ và $Q(x),$ tức là $P(x) | M(x)$ và $Q(x) | M(x)$
    $vi)$ Nếu $M’(x)$ cũng là bội chung của $P(x)$ và $Q(x)$ thì $M’(x)$ cũng là bội của $M(x).$

    Ký hiệu $UCLN$ và $BCNN$ của hai đa thức $P(x)$, $Q(x)$ là $GCD(P(x),Q(x)),$ $LCM(P(x), Q(x))$ hay đơn giản hơn là $(P(x),$ $Q(x)),$ $[P(x), Q(x)].$
    Hai đa thức $P(x),$ $Q(x)$ được gọi là nguyên tố cùng nhau nếu $(P(x), Q(x))= 1.$

    1.8. Thuật toán Euclide

    Để tìm ước chung lớn nhất của hai đa thức $P(x),$ $Q(x),$ ta sử dụng thuật toán Euclide sau đây:

    Định lý 3. Giả sử có hai đa thức $P(x),$ $Q(x),$ trong đó $deg(P)\geq deg(Q).$ Thực hiện phép chia $P(x)$ cho $Q(x)$ được thương số là $S(x)$ và dư số là $R(x).$ Khi đó
    Nếu $R(x)=0$ thì $(P(x),Q(x))=q{*-1}Q(x),$ trong đó $q^*$ là hệ số cao nhất của đa thức $Q(x)$
    Nếu $R(x)\neq 0$ thì $(P(x),Q(x))=(Q(x),R(x))$

    Chứng minh: Nếu $R(x)=0$ thì $P(x)=Q(x).S(x).$ Khi đó đa thức $q^{*-1}Q(x)$ rõ ràng thoả mãn tất cả các điều kiện của $UCLN.$

    Nếu $R(x)\neq 0,$ đặt $D(x)=(P(x),Q(x)),$ $D'(x)=(Q(x),R(x)).$ Ta có $D(x)|P(x)-Q(x).S(x)=R(x),$ suy ra là ước chung của $Q(x),$ $R(x),$ theo định nghĩa của $D'(x),$ ta có $D'(x)$ là ước của $D(x).$ Mặt khác $D’(x)|Q(x)S(x)+R(x)= P(x),$ suy ra $D’(x)$ là ước chung của $P(x),$ $Q(x),$ theo định nghĩa của $D(x),$ ta có $D(x)$ là ước của $D’(x).$ Từ đây, do $D$ và $D’$ đều là các đa thức đơn khởi, ta suy ra $D=D’.$

    Định lý trên giải thích cho thuật toán Euclide để tìm $UCLN$ của hai đa thức theo như ví dụ dưới đây:

    Ví dụ: Tìm ước chung lớn nhất của hai đa thức $x^5-5x+4$ và $x^3-3x^2+2$

    Ta lần lượt thực hiện các phép chia

    $x^5-5x+4$ cho $x^3-3x^2+2$ được $x^2+3x+9$ dư $25x^2-11x-14$

    $x^3 – 3x^2 + 2$ cho $25x^2 –11x – 14$ được $\frac{25x – 64}{625}$, dư $\frac{354}{625}(x-1)$
    $25x^2 – 11x – 14$ cho $x-1$ được $25x + 14$ dư $0$
    Vậy $(x^5 – 5x + 4, x^3 – 3x^2 + 2) = x – 1.$
    Lưu ý, trong quá trình thực hiện, ta có thể nhân các đa thức với các hằng số khác $0.$ Ví dụ trong phép chia cuối cùng, thay vì chia $25x^2 – 11x – 14$ cho $\frac{354}{625}(x-1)$ ta đã chia cho $x – 1.$

    1.9. Tính chất của phép chia hết
    Nhắc lại, hai đa thức $P(x),$ $Q(x)$ được gọi là nguyên tố cùng nhau nếu $(P(x), Q(x))= 1.$ Ta có định lý thú vị và có nhiều ứng dụng sau về các đa thức nguyên tố cùng nhau:

    Định lý 4. (Bezout) Hai đa thức $P(x)$ và $Q(x)$ nguyên tố cùng nhau khi và chỉ khi tồn tại các đa thức $U(x),$ $V(x)$ sao cho $P(x).U(x)+Q(x).V(x)=1.$

    Chứng minh. Giả sử tồn tại các đa thức $U(x)$ và $V(x)$ thoả mãn điều kiện $P(x).U(x)+Q(x).V(x)=1.$
    Đặt $D(x)=(P(x),Q(x))$ thì $D(x)|P(x),D(x)|Q(x)$ suy ra $D(x)|1=P(x).U(x) + Q(x).V(x).$ Suy ra $D(x)=1.$
    Ngược lại, giả sử $(P(x),Q(x)) =1.$ Ta chứng minh tồn tại các đa thức $U(x)$ và $V(x)$ sao cho
    $P(x).U(x)+Q(x).V(x)=1.$ Ta chứng minh bằng quy nạp theo $m=min\begin{Bmatrix} deg(P),deg(Q)\end{Bmatrix}$.
    Nếu $m = 0$ thì điều cần chứng minh là hiển nhiên. Chẳng hạn nếu $deg(Q)=0$ thì $Q=q$ là hằng số và ta chỉ cần chọn $U(x)=0,$ $V(x)=q^{-1}$ thì ta được $P(x).U(x)+ Q(x).V(x)=1.$

    Giả sử ta đã chứng minh định lý đúng đến $m.$ Xét hai đa thức $P(x),$ $Q(x)$ có $min\begin{Bmatrix} deg(P),deg(Q) \end{Bmatrix}=m+1$.
    Không mất tính tổng quát, giả sử $m+1=deg(Q).$ Thực hiện phép chia $P(x)$ cho $Q(x)$ được thương là $S(x)$ và dư là $R(x).$ Không thể xảy ra trường hợp $R(x)=0$ vì khi đó $1=(P(x),Q(x))=q^{*-1}Q(x).$ Vì vậy, ta có $1=(P(x),Q(x))=(Q(x),R(x))$

    Lúc này, do $min(deg(Q),deg(R))=deg(R)<m +1$ nên theo giả thiết quy nạp, tồn tại các đa thức $U*(x),$ $V*(x)$ sao cho $Q(x)V*(x) + R(x)U*(x) = 1.$ Thay $R(x)= P(x)–Q(x).S(x),$ ta được
    $Q(x)V*(x) + (P(x) – Q(x)S(x))U*(x) =1$
    Hay
    $P(x)U*(x) + Q(x)(V*(x) – S(x)U*(x)) =1$
    Đặt $U(x) = U*(x),$ $V(x) = V*(x) – S(x)U*(x)$ ta được đpcm.

    Tính chất của phép chia hết

    $i)$ $Q|P, Q|R$ suy ra $Q|P+R$ hay tổng quát hơn $Q|P.U+R.V$ với $U$, $V$ là các đa thức bất kỳ.
    $ii)$ $Q|P,P|R$ suy ra $Q|R$ (tính bắc cầu)
    $iii)$ $Q|P, P|Q$ suy ra tồn tại số thực khác $0$ $a$ sao cho $Q=aP$ (ta gọi $P$ và $Q$ là các đa thức đồng dạng)
    $iv)$ Nếu $Q_1|P_1$ và $Q_2|P_2$ thì $Q_{1}.Q_{2}|P_{1}.P_{2}$.
    $v)$ Nếu $Q|P.R$ và $(P, Q)=1$ thì $Q|R.$
    $vi)$ Nếu $Q|P,R |P$ và $(Q, R)=1$ thì $Q.R|P$

    Chứng minh. Các tính chất $i-iv)$ là hiển nhiên xuất phát từ định nghĩa $Q|P$ tồn tại $S$ sao cho $P=Q.S.$

    Để chứng minh các tính chất $v)$ và $vi),$ ta sẽ áp dụng định lý Bezout.
    $v)$ Từ giả thiết $Q|P.R$ và $(P,Q) =1$ suy ra tồn tại $S$ sao cho $P.R = Q.S$ và $U,$ $V$ sao cho $P.U+Q.V=1$
    Khi đó $R=(P.U+Q.V).R=(P.R)U+Q.V.R=Q.S.U+Q.V.R=Q.(SU+VR)$ suy ra $Q|R.$
    $vii)$ Từ giả thiết $Q|P, R|P$ và $(Q, R)=1$ suy ra $P=Q.S.$ Vì $P=Q.S$ chia hết cho $R,$ mà $(Q, R)=1$ nên theo $v)$ suy ra $S$ chia hết cho $R,$ tức là $S=R.S1.$ Vậy $P=Q.S =(Q.R).S1$ suy ra $P$ chia hết cho $Q.R.$

    1.10. Các ví dụ có lời giải

    Bài toán 1. Tìm tất cả các cặp số $a,$ $b$ sao cho $x^4+4x^3+ax^2+bx+1$ là bình phương của một đa thức.

    Giải: Nếu $x^4+4x^3+ax^2+bx+1$ là bình phương của một đa thức thì đa thức đó phải có bậc $2.$ Giả sử
    $x^4+4x^3+ax^2+bx+1=(Ax^2+Bx+C)^2$
    $\Leftrightarrow$$x^4+4x^3+ax^2+bx+1=A^2x^4+2ABx^3+(2AC+B^2)x^2+2BCx+C^2$
    Đồng nhất hệ số hai vế, ta được
    $A^2= 1,2AB=4,2AC+B^2=a,2BC=b,C^2 =1.$
    Không mất tính tổng quát, có thể giả sử $A=1,$ suy ra $B=2.$ C có thể bằng $1$ hoặc $-1.$ Nếu $C = 1$ thì $a=6,$ $b=4.$ Nếu $C = -1$ thì $a=2,$ $b=-4.$
    Vậy có hai cặp số $(a, b)$ thoả mãn yêu cầu bài toán là $(6,4)$ và $(2,-4).$

    Bài toán 2. Cho đa thức $P(x)$ và hai số $a, b$ phân biệt. Biết rằng $P(x)$ chia cho $x-a$ dư $A,$ $P(x)$ chia cho $x-b$ dư $B.$ Hãy tìm dư của phép chia $P(x)$ cho $(xa)(xb).$

    Giải: Giả sử $P(x) = (xa)(xb)Q(x) + Cx + D.$ Lần lượt thay $x = a,$ $b,$ ta được
    $A = Ca + D, B = Cb + D$
    Từ đó suy ra $C = \frac{AB}{a-b}, D = A – AB\frac{a}{a-b} = \frac{aB – bA}{a-b}.$​
    Bài toán 3. Tìm dư trong phép chia $x^{100}$ cho $\left ( x-1 \right )^2$.
    Giải: Giả sử $x^{100}=\left ( x-1 \right )^2Q\left ( x \right )+Ax+B$. Thay $x=1$, ta được
    $1 = A + B$.
    Lấy đạo hàm hai vế rồi cho $x = 1$, ta được
    $100 = A$
    Từ đó suy ra dư là $100x – 99$.
     
  2. Tác giả: LTTK CTV
    Đánh giá: ✪ ✪ ✪ ✪ ✪
    PHẦN II: ĐA THỨC VÀ NGHIỆM
    Nghiệm của đa thức đóng một vai trò quan trọng trong việc nghiên cứu các tính chất của đa thức. Nhiều tính chất của đa thức được thể hiện qua nghiệm của chúng. Ngược lại, việc nghiên cứu tính chất các nghiệm của đa thức cũng cũng là một trong các vấn đề trung tâm của đại số.

    2.1. Ví dụ mở đầu
    Xét xem số $\alpha=\sqrt[3]{3+\sqrt{3+\sqrt{3}}}$ là hữu tỉ hay vô tỉ​
    Ta có thể giải bài toán này bằng cách chứng minh lần lượt các mệnh đề sau:
    $1)$ Nếu $a$ vô tỷ thì $\alpha$ vô tỷ
    $2)$ Nếu $a$ vô tỷ thì $\sqrt[3]{\alpha}$ vô tỷ
    $3)$ $\sqrt{3}$ vô tỷ
    Nhưng ta cũng có thể có một cách tiếp cận khác như sau:
    $1)$ Tìm đa thức với hệ số nguyên nhận $\alpha$ làm nghiệm
    $2)$ Chứng minh rằng đa thức này không có nghiệm hữu tỷ
    Việc tìm đa thức với hệ số nguyên nhận $\alpha$ làm nghiệm được tiến hành như sau:
    $\alpha=\sqrt[3]{3+\sqrt{3+\sqrt{3}}}\Rightarrow a^3=3+\sqrt{3+\sqrt{3}}\Rightarrow (a^3-3)^2=3+\sqrt{3}$
    $\Rightarrow ((a^3-3)^2-3)=3$$a^{12}-12a^9+48x^6-72x^3+33=0$ $(*).$
    Vấn đề còn lại là chứng minh $(*)$ không có nghiệm hữu tỷ. Việc này sẽ được thực hiện ở cuối bài.

    2.2. Nghiệm của đa thức, định lý Bezout.
    Định nghĩa.
    Số thực $a$ (trong một số trường hợp, ta xét cả các số phức) được gọi là nghiệm của đa thức $P(x) = a^nx_n + a^{n-1}x_{n-1} +...+ a_1x + a_0$ nếu $P(a)=0,$ tức là
    $$a^na_n + a^{n-1}a_{n-1} +..+ a_1a + a_0 = 0.$$

    Ta có định lý đơn giản nhưng rất có nhiều ứng dụng sau đây về nghiệm của đa thức:

    Định lý 5. $a$ là nghiệm của đa thức $P(x)$ khi và chỉ khi $P(x)$ chia hết cho $x – a.$

    Định lý này là hệ quả của định lý sau:

    Định lý 6. Số dư trong phép chia đa thức $P(x)$ cho $x – a$ là $P(a).$

    Cả định lý $5$ và định lý $6$ đều được gọi là định lý $Bezout.$ Để chứng minh định lý $6,$ ta chỉ cần chứng minh $P(x) – P(a)$ chia hết cho $x – a.$ Nhưng điều này là hiển nhiên vì
    $$P(x)–P(a) = a_n(x^n-a^n) + a_{n-1}(x^{n-1}-a^{n-1}) +...+ a_1(x-a)$$​

    $$x^k-a^k=(x-a)(x^{k-1}+x^{k-2}a+...+a^{k-1})$$​

    Từ định lý $5,$ ta có thể có một định nghĩa khác cho nghiệm của đa thức như sau: $a$ là nghiệm của đa thức $P(x)$ nếu $P(x)=(x-a)Q(x)$ với $Q(x)$ là một đa thức nào đó. Với định nghĩa này, ta có thể phát triển thành định nghĩa về nghiệm bội.

    Định nghĩa. $a$ được gọi là nghiệm bội $r$ của đa thức $P(x)$ nếu $P(x)=(x-a)^{r}Q(x)$ với $Q(a)\neq 0$

    2.3. Định lý Vieta

    Định lý 7. Xét đa thức $P(x) \in R[x].$ Nếu $x_1,x_2,...,x_k$ là các nghiệm phân biệt của $P(x)$ với các bội $r_1,r_2,...,r_k$ tương ứng thì $P(x)$ chia hết cho $(x-x_1)^{r_1}(x-x_2)^{r_2}...(x-x_k)^{r_k}$.

    Chứng minh: Điều này là hiển nhiên theo định nghĩa và do các đa thức $(x-x_i)^{r_i}$ đôi một nguyên tố cùng nhau.

    Hệ quả:
    a) Một đa thức bậc $n$ với hệ số thực có không quá $n$ nghiệm thực.
    b) Nếu hai đa thức $P(x)$ và $Q(x)$ có bậc nhỏ hơn hay bằng $n$ bằng nhau tại $n+1$ điểm thì hai đa thức này bằng nhau.
    $$x^k-a^k=(x-a)(x^{k-1}+x^{k-2}a+...+a^{k-1})$$​
    Định lý 8. Xét đa thức $P(x) \in R[x]$ bậc $n.$ Giả sử $x_1,x_2,...,x_k$ các nghiệm phân biệt của $P(x)$ với các bội $r_1,r_2, …,r_k$ tương ứng. Nếu $r_1+r_2+…+r_k=n$ thì
    $$P(x)=a_n(x-x_1)^{r_1}(x-x_2)^{r_2}...(x-x_k)^{r_{k}}$$​
    Chứng minh: Dùng định lý 9, ta suy ra $P(x)$ chia hết cho $(x-x_1)^{r_1}(x-x_2)^{r_2}…(x-x_k)^{r_k}$, suy ra$P(x)=(x-x_1)^{r_1}(x-x_2)^{r_2}…(x-x_k)^{r_k}Q(x)$. So sánh bậc và hệ số cao nhất, ta suy ra $Q(x)=a_n$.

    Định lý 9. (Định lý $Vieta$) Giả sử đa thức $P(x)=a_nx^{n}+a_{n-1}x^{n-1}+...+a_1x+a_0$ có $n$ nghiệm (trong đó có thể có các nghiệm bội) là $x_1,x_2,..,x_n$ thì $P(x)=a_n(x-x_1)(x-x_2)...(x-x_n)$
    và như hệ quả ta có:
    $$x_1+x_2+...+x_n=\frac{-a_{n-1}}{a_n};$$​

    $$x_1x_2+x_1x_3+...+x_1x_n+x_2x_3+...+x_2x_n+...+x_{n-1}x_n=\frac{a_{n-2}}{a_n};$$
    $$...$$
    $$x_1x_2...x_n=(-1)\frac{na_0}{a_n}.$$​
    Định lý 10. (Định lý $Vieta$ đảo)
    a) Nếu $x+y=S$, $x.y=P$ thì $x,y$ là 2 nghiệm của phương trình
    $$X^2-SX+P=0$$​
    b) Nếu $x+y+z=S$, $xy + yz + zx = T, xyz = P$ thì x, y, z là 2 nghiệm của phương trình
    $$x^3-Sx^2+Tx-P=0$$​
    Từ định lý 8 ta suy ra hai hệ quả đơn giản nhưng rất hiệu quả trong giải toán sau:

    Hệ quả 1. Một đa thức bậc n có không quá n nghiệm.
    Hệ quả 2. Nếu $P(x)$ và $Q(x)$ là các đa thức bậc không quá n, trùng nhau tại n+1
    điểm phân biệt thì hai đa thức này trùng nhau.

    2.4.Bài tập có lời giải


    Bài 1. Cho a, b, c là ba nghiệm của phương trình $x^3-3x+1$= 0. Lập phương trình
    bậc ba có nghiệm là
    a/ $a^2,b^2,c^2$
    b/ $\frac{1-a}{1+a},\frac{1-b}{1+b},\frac{1-c}{1+c}$

    Lời giải:

    Theo định lý $Vieta,$ ta có
    $a+b+c=0$, $ab+bc+ca=-3$, $abc=-1$.
    Từ đó ta tính được
    $a^2 + b^2 + c^2=(a+b+c)^2-2(ab+bc+ca)= 0^2 -2(-3) = 6.$
    $a^2b^2 +b^2c^2 +c^2a^2=(ab+bc+ca)-2abc(a+b+c)=(-3)^2-2.-1.0=9$
    $a^2b^2c^2 = (abc)^2 = 1$
    Áp dụng định lý $Vieta$ đảo, suy ra $a^2, b^2, c^2$ là ba nghiệm của phương trình
    $$x^3-6x^2+9x-1=0$$​
    Tương tự ta tính được:
    $\frac{1-a}{1+a},\frac{1-b}{1+b},\frac{1-c}{1+c}$=$\frac{(1-a)(1+b)(1+c)+(1+a)(1-b)(1+c)+(1+a)(1+b)(1-c)}{(1+a)(1+b)(1+c)}$=$\frac{3+a+b+c-(ab+bc+ca)-3abc}{1+a+b+c+ab+bc+ca+abc}=\frac{-9}{3}=-3$
    $\frac{1-a}{1+a}.\frac{1-b}{1+b}+\frac{1-b}{1+b}.\frac{1-c}{1+c}+\frac{1-c}{1+c}.\frac{1-a}{1+a}=\frac{(1-a)(1-b)(1+c)+(1 +a)(1-b)(1-c)+(1-a)(1+b)(1+c)}{1+a+b+c+ab+bc+ca+abc}=\frac{3-(a+b+c)-(ab+bc+ca)+3abc}{1+a+b+c+ab+bc+ca+abc}=\frac{3}{-3}=-1$
    $\frac{1-a}{1+a}.\frac{1-b}{1+b}.\frac{1-c}{1+c}=\frac{1-(a+b+c)+(ab+bc+ca)-abc}{1+a+b+c+(ab+bc+ca)+abc}=\frac{1}{3}$
    từ đó suy ra $\frac{1-a}{1+a},\frac{1-b}{1+b},\frac{1-c}{1+c}$ là $3$ nghiệm của phương trình;
    $x^3+3x^2-x-\frac{1}{3}=0$
    Bài 2. Rút gọn biểu thức
    $A=\frac{a^2}{(a-b)(a-c)}+\frac{b^2}{(b-c)(b-a)}+\frac{c^2}{(c-a)(c-b)}$
    Xét đa thức :
    $F(x)=$$A=\frac{a^2(x-b)(x-c)}{(a-b)(a-c)}+\frac{b^2(x-c)(x-a)}{(b-c)(b-a)}+\frac{c^2(x-a)(x-b)}{(c-a)(c-b)}$
    Ta có :
    $F(a)=F(b)=F(c)=0$. Nhưng $F(x)$ là đa thức bậc nhỏ hơn hay bằng $2$. Do đó $F(x)$ phải đồng nhất $0$.
    Từ đó suy ra hệ số của $x^2$ của $F(x)$ bằng $0$. . Hệ số này bằng
    $\frac{a^2}{(a-b)(a-c)}+\frac{b^2}{(b-c)(b-a)}+\frac{c^2}{(c-a)(c-b)}-1$
    nên $A=1$

    Bài 3. Tìm tất cả các đa thức $P(x)$ thoả mãn đồng nhất thức $xP(x-1)=(x-26)P(x)$.
    Lời giải.
    Thay $x = 0$ vào đồng nhất thức, ta suy ra $P(0)=0$. Suy ra $P(x)$ chia hết cho $x$, tức là $P(x)=x.P_{1}(x)$. Thay vào đồng nhất thức, ta được
    $x(x-1)P_{1}(x-1)=(x-26)xP_{1}(x)$
    suy ra
    $(x-1)P_{1}(x-1)=(x-26)P_{1}(x)$ $(*)$
    Lại thay $x=1$, ta được $P_{1}$ $(1)=0$, suy ra $P_{1}(x)$ chia hết cho $x-1$, tức là $P_{1}(x)=(x-1)P_{2}(x)$, thay vào $(*)$, ta được
    $(x-1)(x-2)P_{2}(x-1)=(x-26)P_{2}(x-1)(x)$
    Suy ra
    $(x-2)P_{2}(x-1)=(x-26)P_{2}(x)…$
    Cứ tiếp tục lý luận như thế, ta đi đến
    $P(x) = x(x-1)…(x-25)Q(x)$
    Và $Q(x-1)=Q(x)$.

    Đặt $Q(0)=a$ thì ta có $Q(x)=a$ với $x=1,2,3, …$ suy ra $Q(x) = a$ với mọi $x$.
    Vậy $P(x)=ax(x-1)…(x-25)$ là tất cả các nghiệm của bài toán.

    Bài 4. Xét phương trình $x^2–a^{n-1}x^{n-1}–a^{n-1}x^{n-2}-…-a_{1}x–a_{0}=0$ với $a_{i}$ là các số
    thực dương. Chứng minh rằng phương trình này có không quá $1$ nghiệm dương.

    Lời giải. Viết phương trình đã cho dưới dạng
    $1=\frac{a_{n-1}}{x}+\frac{a_{n-2}}{x^2}+...+\frac{a_{0}}{x^n}$
    Vế trái là một hàm số giảm trên (0, $+\infty$ ) nên phương trình trên có không quá 1
    nghiệm dương.

    Bài 5. Với giá trị nào của $A$ và $B$ thì đa thức $P(x)=Ax^{n+1}+Bx^{n+1}$ có $x=1$ là nghiệm bội ít nhất là bậc $2$?

    Lời giải. Trước hết ta phải có $P(1)=0$, tức là $A+B+1=0$, suy ra $B=–A–1$.
    Khi đó $P(x)=Ax^{n-1}–x^{n-1}=(x-1)(Ax^{n}–x^{n-1}–x^{n-2}-… - 1)=(x-1)Q(x)$. Để $1$ là nghiệm bội ít nhất là bậc $2$ thì $Q(x)$ chia hết cho $x-1$, tức là $Q(1)=0$, suy ra $A= n$. Vậy $a=n$, $b=-(n+1)$.


    2.5. Bài tập tự giải

    Bài 1. Biết rằng các nghiệm của phương trình$x^2+ax+b=0$ và $x^2+cx+d=0$
    đều thuộc $(-1, 1)$. Chứng minh rằng các nghiệm của phương trình$2x^2+(a+c)x+(b+d)=0$ cũng thuộc $(-1, 1)$.

    Bài 2. Chứng minh rằng đa thức $P(x)=1+x+x^2/2!+...+x^n/n!$ không có nghiệm bội

    Bài 3. Tìm mối liên hệ giữa các hệ số của phương trình $ax^3+bx^2+cx+d=0$ biết rằng tích của hai nghiệm của phương trình này bằng tổng của chúng.
     
  3. Tác giả: LTTK CTV
    Đánh giá: ✪ ✪ ✪ ✪ ✪
    PHẦN III : ĐA THỨC BẤT KHẢ QUY

    3.1. Đa thức với hệ số nguyên

    Đa thức với hệ số nguyên là đa thức có dạng :
    $$P(x)=a_{n}.x^{n}+a_{n-1}.x^{n-1}+..+a_{1}x+a_{0}$$​
    trong đó $a_{i}$ là các hệ số nguyên
    Ta ký hiệu tập hợp tất cả các đa thức với hệ số nguyên là $Z[x]$
    Ta có các kết quả cơ bản sau đây về đa thức với hệ số nguyên:
    $(1)$ Nếu $P(x)$ có nghiệm nguyên $x=a$ thì phân tích được $P(x)=(x-a).Q(x)$ với $Q(x)$ là đa thức với hệ số nguyên .
    $(2)$ Nếu $a,b$ nguyên và $a\neq b$ thì $P(a)-P(b)\vdots a-b$
    $(3)$ Nếu $x=\frac{p}{q}$ là một nghiêm của $P(x)$ ( với $(p,q)=1$ ) thì $p$ là ước của $a_{0}$ và q là ước của $a_{n}$ . Đặc biệt nếu $a_{n}=\underline{+}1$ thì nghiệm hữu tỷ là nghiệm nguyên
    $(4)$ Nếu $x=m+\sqrt{n}$ là nghiệm của $P(x)$ với $m,n$ nguyên , $n$ không là số chính phương thì ${x}'=m-\sqrt{n}$ cũng là nghiệm của $P(x)$
    $(5)$ Nếu $x=m+\sqrt{n}$ với $m,n$ nguyên , $n$ không là số chính phương thì $P(x)={M}'+{N}'\sqrt{n}$ với ${M}',{N}'$ nguyên
    Đa thức với hệ số nguyên sẽ nhận giá trị nguyên với mọi giá trị x nguyên. Điều ngược lại không đúng, có những đa thức nhận giá trị nguyên với mọi x nguyên nhưng các hệ số của nó không nguyên.
    Ví dụ: Các đa thức $\frac{x^2-x}{2}$, $\frac{x^3-x}{6}$ nhận giá trị nguyên với mọi $x$ nguyên
    Đa thức với hệ số hữu tỷ nhưng nhận giá trị nguyên với mọi x nguyên được gọi là đa thức nguyên.
    Một đa thức với hệ số hữu tỷ $P(x)$ bất kỳ có thể biểu diễn được dưới dạng $\frac{a}{b}Q(x)$ với $a,b$ là các số nguyên và $Q(x)$ cũng là đa thức với hệ số nguyên
    3.2. Đa thức bất khả quy
    Định nghĩa:
    Cho $P(x)$ là đa thức với hệ số nguyên. Ta gọi $P(x)$ là bất khả quy trên $Z[x]$ với bậc lớn hơn hay bằng 1
    Tương tự định nghĩa đa thức bất khả quy trên $Q[x]$
    Định lý 3.1 (Tiêu chuẩn Eisenstein)
    Cho:
    $$P(x)=a_{n}x^{n}+a_{n-1}x^{n-1}+...+a_{1}x+a_{0}$$​
    Nếu tồn tại số nguyên tố $p$ sao cho
    i) $a_{n}$ không chia hết cho $p$
    ii) $a_{0},a_{1},..,a_{n-1}$ chia hết cho $p$
    iii) $a_{0}$ không chia hết cho $p^2$
    thì đa thức $P(x)$ bất khả quy
    Định lý 3.2 (Quan hệ bất khả quy trên $Z[x]$$Q[x]$)
    Nếu đa thức $P(x)$ $\epsilon$ $Z[x]$ bất khả quy trên $Z[x]$ thì cũng bất khả quy trên $Q[x]$.
    Bổ đề Gauss. Ta gọi đa thức $P(x)$ $\epsilon$ $Z[x]$ là nguyên bản nếu các hệ số của nó nguyên tố cùng nhau.
    Ta có bổ đề Gauss: Tích của hai đa thức nguyên bản là nguyên bản.
    Chứng minh bổ đề. Cho hai đa thức nguyên bản:
    $$P(x)=a_{n}x^{n}+a_{n-1}x^{n-1}+...+a_{1}x+a_{0}$$​

    $$Q(x)=b_{m}x^{m}+b_{m-1}x^{m-1}+...+b_{1}x+b_{0}$$​
    thì
    $$P(x).Q(x)=c_{m+n}x^{m+n}+c_{m+n-1}x^{m+n-1}+...+c_{1}x+c_{0}$$​
    Giả sử tích trên không nguyên bản thì tồn tại một số nguyên tố $p$ là ước chung của các hệ số $c_{0},c_{1},...,c_{m+n}$. Vì $P(x)$ nguyên bản nên gọi $i$ là số nhỏ nhất mà $a_{i}$ không chia hết cho $p$ và $j$ là số nhỏ nhất sao cho $bj$ không chia hết cho $p$. Khi đó xét $x_{i+j}$ ta thấy hệ số tương ứng không chia hết cho $p$, vô lý .
    Vậy tích trên nguyên bản
    Chứng minh định lý: Cho $P(x)$ bất khả quy trên $Z[x]$.Giả sử $P(x)$ khả quy trên $Q[x]$: $P(x)$$=P_{1}(x).P_{2}(x)$
    với $P_{1},P_{2}$ là đa thức bậc nhỏ hơn bậc của P và có hệ số hữu tỷ .
    Đặt :
    $$P_{1}(x)=\frac{a_{1}}{b_{1}}Q_{1}(x),P_{2}(x)=\frac{a_{2}}{b_{2}}Q_{2}(x)$$​
    với $(a_{i},b_{i})=1$ và $Q_{i}$ nguyên bản $(i_{1}=1,2)$
    Khi đó :
    $$P(x)=\frac{a_{1}a_{2}}{b_{1}b_{2}}Q_{1}(x)Q_{2}(x)=\frac{p}{q}Q_{1}(x)Q_{2}(x)$$​
    với $(p,q)=1$
    Do $P(x)\epsilon Z[x]$ nên từ đây suy ra các hệ số của $Q_{1}(x),Q_{2}(x)$ đều chia hết cho $q$, suy ra $Q_{1}(x),Q_{2}(x)$ không nguyên bản, trái với bổ đề Gauss . Mâu thuẫn. Vậy $P(x)$ bất khả quy trên $Q[x].$
    3.3. Một số tính chất của đa thức bất khả quy
    3.4. Một số bài tập có lời giải
    Bài 1
    . Cho tam thức bậc hai $P(x)=ax^2+bx+c$ với $a,b,c$ là các số hữu tỷ.
    Chứng minh rằng $P(x)$ nguyên với mọi $x$ nguyên khi và chỉ khi $c, a+b$ và $2a$ nguyên.
    Bài 2. a) Tìm tất cả các số nguyên $a$ sao cho $(x-a)(x-10)+1$ có thể phân tích được thành tích dạng $(x+b)(x+c)$ với $b, c$ là các số nguyên.
    b) Tìm tất cả các số nguyên khác $0$ và đôi một khác nhau $a,b,c$ sao cho đa thức
    $$x(x-a)(x-b)(x-c)+1$$​
    có thể biểu diễn dưới dạng tích của hai đa thức với hệ số nguyên.
    Bài 3. Chứng minh các đa thức sau là bất khả quy
    $$a)x^3+5x^2+35$$
    $$b)x^4-x^3+2x+1$$
    Bài 4. Cho $p$ là số nguyên tố. Chứng minh rằng đa thức:
    $$x^{p-1}+x^{p-2}+...+x+1$$​

    bất khả quy.​
    Bài 5. Cho $n$ số $a_{i}$ thuộc $\mathbb{Z}$. Chứng minh:
    a) $(x-a_{1})(x-a_{2})…(x-a_{n})–1$ bất khả quy
    b) $(x-a_{1})^2(x-a_{2})^2…(x-an)^2+1$ bất khả quy
    3.4. Bài tập
    Bài 1
    . Đa thức $P(x)$ bậc $n$ có hệ số hữu tỷ là đa thức nguyên khi và chỉ khi nó nhận giá trị nguyên tại $n+1$ điểm nguyên liên tiếp. Chứng minh.
    Bài 2. Tìm tất cả các giá trị $n$ sao cho tồn tại $n$ số nguyên phân biệt $a_{1},a_{2},...,a_{n}$ để $(x-a_{1})(x-a_{2})…(x-a_{n})+1$ khả quy.
    Bài 3. (Tiêu chuẩn Eisenstein mở rộng) Cho đa thức:
    $$P(x)= a_{n}x^n + a_{n-1}x^{n-1} + …+ a_{1}x + a_{0}$$​
    Giả sử tồn tại số nguyên tố p sao cho:
    i) $a_{n}$ không chia hết cho $p$
    ii) $a_{0}$ không chia hết cho$p^2$
    iii)$a_{0},a_{1},...,a_{n-k}\vdots p$
    Khi đó nếu $P(x) = H(x)$.$G(x)$ thì một trong hai đa thức $H(x), G(x)$ có bậc nhỏ hơn $k.$
    Bài 4. Tìm tất cả các giá trị $n$ nguyên dương sao cho đa thức $x^n+4$ khả quy trên $Z[x]$.
    Bài 5. Chứng minh rằng với mọi số nguyên dương $n$, đa thức $x^n + 5x^{n-1} + 3$ bất khả quy.
    Bài 6. Tìm hệ số tự do của đa thức $P(x)$ với hệ số nguyên, biết rằng trị tuyệt đối của nó nhỏ hơn 1000 và $P(19) = P(94) = 1994$.
     
  4. Tác giả: LTTK CTV
    Đánh giá: ✪ ✪ ✪ ✪ ✪
    PHẦN IV: CÔNG THỨC NỘI SUY LAGRANGE

    4.1. Các ví dụ mở đầu
    Ví dụ 1.
    Tìm tất cả các đa thức thoả mãn điều kiện $P(x)$ thoả điều kiện
    $P(1)=1;P(2)=2;P(3)=4$​
    Lời giải: Rõ ràng nếu $P$ và $Q$ là hai đa thức thoả điều kiện của để bài thì $P(x)-Q(x)$ sẽ bằng $0$ tại các điểm $1,2,3$ và từ đó , ta có $P(x)-Q(x)=(x-1)(x-2)(x-3).H(x)$ . Ngược lại nếu $P(x)$ là đa thức thoả mãn điều kiện của để bài thì các đa thức $Q(x)=P(x)+(x-1)(x-2)(x-3).H(x)$ cũng thoả mãn điều kiện đề bài với mọi $H(x).$ Từ đó có thể thấy rằng có vô số các đa thức thoả mãn điều kiện đề bài.
    Ta đặt ra câu hỏi: Trong các đa thức thoả mãn điều kiện đề bài, hãy tìm đa thức có bậc nhỏ nhất. Rõ ràng đa thức này không thể là hằng số, cũng không thể là bậc nhất. Ta thử tìm bậc tiếp theo là bậc $2.$
    Giả sử $P(x) = ax^2 + bx + c$ là đa thức thoả mãn điều kiện đề bài. Khi đó:
    $P(1)=1\Rightarrow a+b+c=1$
    $P(2)=2\Rightarrow 4a+2b+c=2$
    $P(3)=4\Rightarrow 9a+3b+c=4$
    Giải hệ này ra ta được $\left ( a,b,c \right )=\left ( \frac{1}{2};\frac{-1}{2};1 \right )$ , ta được $P(x)=\frac{1}{2}x^2-\frac{1}{2}x+1$ là đa thức bậc nhỏ nhất thoả mãn điều kiện. Và theo như lý luận ở trên , mọi nghiệm của bài toán sẽ có dạng:
    $Q(x) = P(x) + (x-1)(x-2)(x-3)H(x)$
    với $H(x)$ là một đa thức tuỳ ý.​
    Ví dụ 2. Tìm đa thức bậc nhỏ nhất thoả mãn điều kiện
    $P(-2) = 0, P(-1) = 1, P(0) = 1, P(1) = 2, P(2) = 3$​
    Lời giải. Từ ý tưởng phương pháp hệ số bất định và hệ phương trình bậc nhất ở trên. Ta thấy rằng chắn chắn sẽ tồn tại đa thức bậc không quá $4$ thoả mãn điều kiện đề bài. Xét $P(x) = ax^4 + bx^3 + cx^2 + dx + e$. Từ điều kiện đề bài suy ra hệ:
    $\left\{\begin{matrix} 16a-8b+4c-2d+e=0\\ a-b+c-d+e=1\\ e=1\\ a+b+c+d+e=2\\ 16a+8b+4c+2d+e=3 \end{matrix}\right.$
    Giải hệ này ta được $a = \frac{-1}{8}, b =\frac{-1}{12}, c = \frac{5}{8}, d = \frac{5}{12}, e = 1.$

    4.2. Công thức nội suy Lagrange

    Từ các ví dụ cụ thể nêu trên, ta có thể dự đoán rằng với mọi các bộ $n+1$ số phân biệt $\left ( a_{0},a_{1},...,a_{n} \right )$ và bộ $n+1$ số bất kỳ $b_{0}, b_{1}, ..., b_{n}$ sẽ tồn tại một đa thức $P(x)$ bậc không vượt quá $n$ thoả mãn điều kiện
    $P(a_{i}) = bi ,\forall i=0, 1, 2, ..., n. (*)$​
    Ngoài ra, do tất cả các đa thức $Q(x)$ thoả mãn $(*)$ sẽ phải có dạng
    $Q(x) = P(x) +(x-a_{0})(x-a_{1})...(x-a_{n}).H(x)$​
    với $H(x)$ là một đa thức nào đó nên các nghiệm khác của $(*)$ đều có bậc $\geq n+1.$
    Vì thế ta có thể đề xuất định lý sau:
    Định lý. Cho bộ $n+1$ số thực phân biệt (a_{0}, a_{1}, ..., a_{n}) và bộ $n+1$ số bất kỳ (b_{0}, b{1},..., b_{n}). Khi đó tồn tại duy nhất một đa thức $P(x)$ có bậc không vượt quá $n$ thoả mãn điều kiện $P(a_{i}) = bi ,\forall i=0, 1, 2, ..., n. (*)$
    Sự duy nhất được chứng minh khá dễ dàng theo như lý luận ở trên. Tuy nhiên, việc chứng minh tồn tại cho trường hợp tổng quát là không đơn giản, vì điều này tương đương với việc chứng minh một hệ phương trình $n+1$ phương trình, $n+1$ ẩn số có nghiệm (duy nhất). Rất thú vị là ta tìm được cách chứng minh định lý này một cách xây dựng, tức là tìm ra được biểu thức tường minh của đa thức P(x) mà không cần phải giải hệ phương trình hệ số bất định nêu trên.
    Ý tưởng chứng minh này như sau. Ta đi tìm các đa thức $P_{0}(x), P_{1}(x) …, P_{n}(x)$ bận $n$ thoả mãn điều kiện sau:
    $P_{i}(a_{j})=\delta _{ij}$​
    Trong đó :

    $\delta _{ij}=\left\{\begin{matrix} 1 i=j\\0 i\neq j \end{matrix}\right.$​
    Khi đó đa thức $P(x)=\sum_{i=0}^{n}b_{i}P_{i}(x)$ sẽ thoả mãn điều kiện vì
    $P(a_{j})=\sum_{i=0}^{n}b_{i}P_{i}(a_{j})=\sum_{i=0}^{n}b_{i}\delta _{ij}=b_{j}$​
    Vấn đề còn lại là đi tìm các đa thức $P_{i}(x)$. Vì $P_{i}(a_{j})=0$ với mọi $i\neq j$ nên:
    $P_{i}(x)=C_{i}(x-a_{0})...(x-a_{i-1})(x-a_{i+1})...(x-a_{n})$​
    Vì $P_{i}(a_{i})=1$ nên
    $C_{i}=\frac{1}{(a_{i}-a_{0})...(a_{i}-a_{i-1})(a_{i}-a_{i+1})...(a_{i}-a_{n})}$​
    Như thế ta tìm được
    $P_{i}(x)=\frac{(x-a_{0})...(x-a_{i-1})(x-a_{i+1})...(x-a_{n})}{(a_{i}-a_{0})...(a_{i}-a_{i-1})(a_{i}-a_{i+1})...(a_{i}-a_{n})} (**) $
    là các đa thức thỏa mãn hệ điều kiện $P_{i}(a{j})=\delta_{ij}$​
    Công thức nội suy Lagrange. Cho bộ $n+1$ số thực phân biệt $(a_{0}, a_{1}, ..., a_{n})$ và bộ $n+1$ số bất kỳ $(b_{0}, b_{1}, ..., b_{n})$. Khi đó đa thức
    $P(x)=\sum_{i=0}^{n}b_{i}P_{i}(x)$​
    là đa thức duy nhất có bậc không vượt quá $n$ thỏa mãn điều kiện $P(a_{i})=b_{i}$ với mọi $i=0,1,2,3,...,n$. Các đa thức $P_{i}(x)$ là các đa thức bậc $n$ được định nghĩa bởi $(**)$

    4.3. Ứng dụng của công thức nội suy Lagrange

    Bài toán nội suy là một trong các bài toán cơ bản của toán lý thuyết và toán ứng dụng. Trong thực tế, chúng ta không thể đo được giá trị của một hàm số tại mọi điểm, mà chỉ đo được tại một số điểm. Các công thức nội suy cho phép chúng ta, bằng phép đo tại một số điểm, "dựng" lại một đa thức xấp xỉ cho hàm số thực tế. Công thức nội suy Lagrange, vì thế có nhiều ứng dụng trong vật lý, trắc địa, kinh tế học, khí tượng thuỷ văn, dự đoán dự báo … Tuy nhiên, ta sẽ không đi sâu về các vấn đề này. Dưới đây ta xem xét một số ứng dụng của công thức nội suy $Lagrange$ trong các bài toán phổ thông.

    4.4. Các bài tập có lời giải
    Bài 1.
    Rút gọn biểu thức
    $$A=\frac{a^2}{(a-b)(a-c)}+\frac{b^2}{(b-c)(b-a)}+\frac{c^2}{(c-b)(c-a)}$$​
    Lời giải. Áp dụng công thức nội suy $Lagrange$ cho hàm số $P(x) = x^2$ với các điểm $a, b, c$ và giá trị tương ứng là $a^2, b^2, c^2$, ta có:
    $P(x)=\frac{a^2(x-b)(x-c)}{(a-b)(a-c)}+\frac{b^2(x-c)(x-a)}{(b-c)(b-a)}+\frac{c^2(x-a)(x-b)}{(c-b)(c-a)}$​
    So sánh hệ số $x^2$ ở hai vế , ta được $A=1$
    Bài 2. Cho đa thức $P(x)$ bậc $n$ thoả mãn điều kiện $P(k)=\frac{k}{k+1};\forall k=0,1, 2, …, n$. Hãy tìm $P(n+1)$
    Lời giải. Theo công thức nội suy $Lagrange$ thì

    $P(x)=\sum_{k=0}^{n}\frac{k}{k+1}.\frac{x(x-1)...(x-k+1)(x-k-1)...(x-n)}{k(k-1)...1.(-1)...(k-n)}$​
    Từ đó :
    $$P(x)=\sum_{k=0}^{n}\frac{k}{k+1}.\frac{(n+1)...(n-k+2)(n-k)...(1)}{k(k-1)...1.(-1)...(k-n)}=\sum_{k=0}^{n}\frac{k}{k+1}.\frac{(n+1)...(n-k+2)(n-k+1)(n-k)...(1)}{k(k-1)...1.(-1)...(k-n)(n-k+1)}=\sum_{k=0}^{n}(-1)^{n-k}.k.\frac{(n+1)!}{(k+1)!(n-k+1)!}=\frac{1}{n+2}\sum_{k=0}^{n}.(-1)^{n-k}.C_{n+2}^{k+1}$$

    Cách 2. Xét đa thức $(x+1)P(x) – x$ có bậc $n$ và có $n+1$ nghiệm $x = 0, 1, 2, …, n.$
    Do đó, ta có:
    $$(x+1)P(x) – x = ax(x-1)(x-2)…(x-n)$$
    với $a$ là $1$ hằng số.​
    Thay $x=-1$ , ta được $1=a(-1)(-2)...(-n-1)=a(-1)^{n+1}(n+1)!$
    Suy ra $a=\frac{(-1)^{n+1}}{(n+1)!}$
    Từ đó $(n+2)P(n+1)-(n+1)=n!\frac{(-1)^{n+1}}{(n+1)!}=\frac{(-1)^{n+1}}{n+1}$
    Suy ra $P(n+1)=\frac{\left [ (n+1)^2+(-1)^{n+1} \right ]}{(n+2)}$
    Bài 3. Cho tam thức bậc hai $P(x) = ax^2 + bx + c$ thoả mãn điều kiện $\left | P(x) \right |\leq 1,\forall x\leq 1$.Chứng minh rằng:
    $$\left | a \right |+\left | b \right |+\left | c \right |\leq 3$$​
    Lời giải. Thực hiện phép nội suy tại $3$ điểm $-1, 0, 1$ , ta có:
    $P(x)=P(-1).\frac{x(x-1)}{(-1-0)(-1-1)}+P(0).\frac{(x+1)(x-1)}{(0+1)(0-1)}+P(1).\frac{x(x+1)}{(1+0)(1+1)}$
    Suy ra $P(x)=\frac{P(1)+P(-1)-2P(0)}{2}.x^2+\frac{P(1)-P(-1)}{2}x+P(0)$
    Từ đó $a=\frac{P(1)+P(-1)-2P(0)}{2},b=\frac{P(1)-P(-1)}{2},c=P(0)$
    Suy ra
    $$\left | a \right |+\left | b \right |+\left | c \right |=\left | \frac{P(1)+P(-1)-2P(0)}{2} \right |+\left | \frac{P(1)-P(-1)}{2} \right |+\left | P(0) \right |\leq \left | \frac{P(1)+P(-1)}{2} \right |+\left | \frac{P(1)-P(-1)}{2} \right |+2\left | P(0) \right |\leq max\begin{Bmatrix} {\left | P(1) \right |,\left | P(-1) \right |} \end{Bmatrix}++2\left | P(0) \right |\leq 3$$​
    4.5. Bài tập tự giải
    Bài 1.
    Rút gọn biểu thức:
    $$A=\frac{a^4}{(a-b)(a-c)}+\frac{b^4}{(b-a)(b-c)}+\frac{c^4}{(c-a)(c-b)}$$​

    Bài 2. Cho $M(y)$ là một đa thức bậc $n$ sao cho $M(y) = 2^y$ với $y = 1, 2, …, n+1$. Hãy tìm $M(n+2).$
    Bài 3. Cho đa thức $P(x)=x^{10}+a_{9}x^9+...+a_{1}x+a_{0}$. Biết rằng $P(-1)=P(1),P(2)=P(-2),...,P(-5)=P(5)$ . Chứng minh rằng $P(x)=P(-x), \forall x \in \mathbb{R}$
    Bài 4. Cho $x_{0}< x_{1}< x_{2}<...< x_{n}$ là các số nguyên và $P(x)$ là đa thức bậc $n$ có hệ số cao nhất bằng $1$ . Chứng minh rằng tồn tại $i\in\begin{Bmatrix} 0,1,...,n \end{Bmatrix}$ sao cho $\left | P(x_{i}) \right |\geq \frac{n!}{2^{n}}$
    Bài 5. Một chiếc tàu với vận tốc không đổi đi ngang qua một hòn đảo. Thuyền trưởng cứ mỗi giờ lại đo khoảng cách từ tàu đến đảo. Vào lúc $12, 14$ và $15$ giờ tàu cách đảo các khoảng cách tương ứng là $7, 5$ và $11$ km. Hỏi vào lúc $13$ giờ tàu cách đảo bao nhiêu $km$. Và lúc $16$ giờ, tàu sẽ cách đảo bao nhiêu $km$?
    Bài 6. Trên mặt phẳng cho $100$ điểm. Biết rằng với bốn điểm bất kỳ trong chúng đều có một $parabol$ bậc $2$ đi qua. Chứng minh rằng tất cả các điểm đã cho đều nằm trên một $parabol$ bậc 2.
     
  5. Tác giả: LTTK CTV28
    Đánh giá: ✪ ✪ ✪ ✪ ✪
    Bài 7: Tìm tất các các giá trị của số tự nhiên $n$ sao cho đa thức: $P_n(x)=x^{2n}+(x+1)^{2n}+1$ chia hết cho đa thức $T(x)=x^2+x+1$
    Do $x^{4n}-(x+1)^{2n}$ chia hết cho $x^2+x+1$ nên ta cần tìm $n$ sao cho $Q_n(x)=x^{4n}+x^{2n}+1=\left(x^{2n}-x^{n}+1\right)\left(x^{2n}+x^{n}+1\right)$ chia hết cho $x^2+x+1$
    Mặt khác ta có kết quả quen thuộc $x^{3m+1}+x^{3n+2}+1$ chia hết cho $x^2+x+1$ nên nếu $n\equiv 1,2 \bmod 3$ thì $x^{2n}+x^{n}+1$ chia hết cho $x^2+x+1$, suy ra $P_n(x)$ chia hết cho $x^2+x+1$
    Xét $n=3k\ (k\in \mathbb{N})$, ta có $Q_n(x)-3 = \left(x^{2n}-1\right)\left(x^{2n}+2\right)=\left(\left(x^{3}\right)^{2k}-1\right)\left(x^{6k}+2 \right)$
    Suy ra $Q_n(x)-3$ chia hết cho $x^2+x+1$ với $3\mid x$. Như vậy $Q_n(x)$ và $P_n(x)$ không chia hết cho $x^2+x+1$
    Vậy các $n$ thỏa đề bài là những số tự nhiên không chia hết cho 3.
    Bài 8: Tìm tất cả các đa thức có hệ số thực thỏa mãn: $P(x).P(x+1)=P(x^2+2),\forall x\in \mathbb{R}$.
    Với đa thức hằng $P(x)\equiv a$, ta có: $a^2=a$ nên $a=0$ hoặc $a=1$.
    Ta thấy $P(x)\equiv 0$ và $P(x)\equiv 1$ thỏa mãn bài ra.
    Xét trường hợp $P(x)$ khác hằng số. Ta có các nhận xét sau:
    + Đa thức bậc hai $P(x)=x^2-x+2$ thỏa mãn bài ra.
    +Nếu $P(x)$ thỏa mãn bài toán thì hiển nhiên mọi đa thức dạng: $Q(x)=(P(x))^n(n\in \mathbb{N^*})$ cũng thỏa mãn bài ra.
    +Đa thức $P(x)$ thỏa mãn bài toán thì hệ số bậc cao nhất bằng $1$.
    + Đa thức $P(x)$ thỏa mãn bài toán thì nó không thể có nghiệm thực. Thật vây, nếu $x_0$ là nghiệm thì dãy số tăng $x_1=x_0^2+2,x_2=x_1^2+2,...$ cũng là nghiệm, mâu thuẩn.
    Vậy bậc của $P(x)$ là một số chẵn, $degP(x)=2m(m\in \mathbb{N^*})$, đặt
    $P(x)=(x^2-x+2)^m+Q(x),degQ(x)=q<2m$.
    Thế vào điều kiện bài ra, ta thu được:
    $(x^2-x+2)^mQ(x)+(x^2-x+2)^mQ(x+1)+Q(x)Q(x+1)=Q(x^2+2)(1)$.
    Ta chứng minh $Q(x)\equiv 0$.
    Thật vậy, giá sử $Q(x)$ không đồng nhất bằng $0$.
    Ta thấy rằng: Bậc của đa thức ở vế trái $(1)$ bằng $2m+q$, còn bậc ở vế phải bằng $2q$.
    So sánh bậc của hai vế, ta thu được: $q=2m$. Điều này là vô lý.
    Vậy $Q(x)\equiv 0$ và $P(x)=(x^2-x+2)^m$.
    Kết luận: Các đa thức thỏa mãn điều kiện bài toán là $P(x)\equiv 0,P(x)\equiv 1,P(x)=(x^2-x+2)^m,m\in \mathbb{N^*}$
    Bài 9: Hỏi có tất cả bao nhiêu đa thức $P_n(x)$ bậc $n$ chẵn thỏa mãn các điều kiện:
    1) Các hệ số của $P_n(x)$ thuộc tập hợp $M=\left\{0,-1,1\right\}$ và $P_n(0)\ne 0$.
    2) Tồn tại đa thức $Q(x)$ có các hệ số thực thuộc $M$ sao cho $P_n(x)\equiv (x^2-1)Q(x)$.
    Đặt $n=2m$.
    Đa thức $Q(x)$ có bậc $n-2$:
    $Q(x)=b_0x^{n-2}+b_1x^{n-3}+...+b_{n-3}x+b_{n-2}$.
    Do đó:
    $P_n(x)=(x^2-1)Q(x)=b_0x^{n}+b_1x^{n-1}+(b_2-b_0)x^{n-2}+(b_3-b_1)x^{n-3}+...+(b_{n-2}-b_{n-4})x^2-b_{n-3}x-b_{n-2}.$
    Như vậy số các đa thức $P_n(x)$ thỏa mãn đề bài bằng số các dãy số: $(b_0,b_1,...,b_{n-3},b_{n-2})$ thỏa mãn điều kiện:
    $b_i\in M$ với mọi $0\le i\le n-2(*)$.
    $b_{i+2}-b_i\in M$ với mọi $0\le i\le n-4$ và $b_{n-2}\ne 0(b_{n-2}=-P_n(0))$.
    Chú ý: $b_i=1$ thì $b_{i+2}\in \left\{0;1\right\},b_i=-1$ thì $b_{i+2}\in \left\{-1,0\right\},b_i=0$ thì $b_{i+2}\in \left\{-1,0,1\right\}$.
    Ta kí hiệu: $x_k$ là số dãy $(b_0,b_1,...,b_{2k})$ thỏa mãn $b_{2i}\in M$ với mọi $0\le i\le k,b_0\ne 0$ và $b_{i+2}-b_i\in M\forall i=0,2,...,2k-2$;
    $y_{k-1}$ là số dãy $(b_1,b_3,...,b_{2k-1})$ thỏa mãn $b_{2i-1}\in M$ với mọi $1\le i\le k,b_{i+2}-b_i\in M\forall i=1,3,...,2k-3$.
    Ta có: $x_0=2,x_1=4$ và $x_{k+1}=2x_k+x_{k-1}$ vì nếu $b_{2k}\ne 0$ thì $b_{2k+2}\in \left\{0,b_{2k}\right\}$ ,nếu $b_{2k}=0$ thì có ba cách lấy $b_{2k+2}\in M$, số dãy $(b_0,b_2,...,b_{2k})$ mà $b_{2k}=0$ bằng $x_{k-1}$.
    Chứng minh bằng quy nạp theo $k$ ta được:
    $x_k=\frac{(1+\sqrt{2})^{k+1}-(1-\sqrt{2})^{k+1}}{\sqrt{2}}$.
    (dự đoán công thức trên bằng cách dùng phương trình đặc trưng).
    Hoàn toàn tương tự có: $y_0=3,y_1=7.y_{k+1}=2y_k+y_{k-1}\forall k\ge 1$.
    $y_{k-1}=\frac{(1+\sqrt{2})^{k+1}+(1-\sqrt{2})^{k+1}}{2}$.
    Do đó số các dãy $(b_0,b_1,...,b_{2m-3},b_{2m-2})$ thỏa mãn $(*)$ bằng:
    $(x_{m-1}-x_{m-2})y_{m-2}=((1+\sqrt{2})^{m-1}+(1-\sqrt{2})^{m-1}).\frac{1}{2}.((1+\sqrt{2})^m+(1-\sqrt{2})^{m})$.
    $\frac{(1+\sqrt{2})^{2m-1}+(1-\sqrt{2})^{2m-1}+2.(-1)^{m-1}}{2}$
    $=\frac{(1+\sqrt{2})^{n-1}+(1-\sqrt{2})^{n-1}}{2}+(-1)^{\frac{n}{2}-1}$.
    Đó chính là số các đa thức $P_n(x)$ cần tìm.
    Bài 10: Xác định tất cả các đa thức với hệ số thực $P(x),Q(x)$ và $R(x)$ thỏa mãn điều kiện: $\sqrt{P(x)}-\sqrt{Q(x)}=R(x)$ với mọi số thực $x$.
    Ta viết điều kiện bài ra dưới dạng:
    $\sqrt{P(x)}=\sqrt{Q(x)}+R(x)$.
    $\iff \left\{\begin{array}{I} P(x)\ge 0,Q(x)\ge 0,\sqrt{Q(x)}+R(x)\ge 0,\forall x\in \mathbb{R}\\ P(x)=Q(x)+(R(x))^2+2R(x)\sqrt{Q(x)},\forall x\in \mathbb{R}\end{array}\right.$
    Từ đó : $R(x)\sqrt{Q(x)}$ là một hàm đa thức. Vậy $R(x)\equiv 0$ hoặc $Q(x)=(M(x))^2$ với $M(x)$ là một đa thức.
    + Trường hợp $R(x)\equiv 0$, từ điều kiện:
    $\sqrt{P(x)}=\sqrt{Q(x)}+R(x)$ suy ra $P(x)\equiv Q(x)$ ta thu được:
    $(P(x),Q(x),R(x))=(0,T(x),T(x))$ với $T(x)$ là đa thức tùy ý, là nghiệm của bài toán.
    + Trường hợp $Q(x)\equiv (M(x))^2$. Thế vào điều kiện bài toán, ta thu được:
    $\sqrt{P(x)}=M(x)+R(x)$
    hay $P(x)=(M(x)+R(x))^2,\forall x\in \mathbb{R}$.
    Vậy $P(x)\equiv (N(x))^2$, trong đó $N(x)$ à một đa thức. Từ các điều kiện $Q(x)\equiv (M(x))^2$ và $P(x)\equiv (N(x))^2$, ta thu được: $R(x)=N(x)-M(x),$ tức là:
    $(P(x),Q(x),R(x))=(N(x)^2,(M(x))^2,N(x)-M(x))$,
    với $M(x),N(x)$ là các đa thức tùy ý, thỏa mãn điều kiện bài toán ra
     
    Chỉnh sửa cuối: 25/5/19
  6. Tác giả: LTTK CTV28
    Đánh giá: ✪ ✪ ✪ ✪ ✪
    Bài 11: Cho đa thức với hệ số thực $P(x)=x^n+a_{n-1}x^{n-1}+...+a_1x+a_0$ và $Q(x)=x^2+x+2009$. Biết rằng đa thức $P(x)$ có $n$ nghiệm thực phân biệt và đa thức $P(Q(x))$ không có nghiệm thực. Chứng minh rằng: $P(2009)>\frac{1}{4^n}$.
    Cho đa thức với hệ số thực $P(x)=x^n+a_{n-1}x^{n-1}+...+a_1x+a_0$ và $Q(x)=x^2+x+2009$. Biết rằng đa thức $P(x)$ có $n$ nghiệm thực phân biệt và đa thức $P(Q(x))$ không có nghiệm thực. Chứng minh rằng: $P(2009)>\frac{1}{4^n}$.
    Do $P(x)$ có $n$ nghiệm thực phân biệt $x_1,x_2,...,x_n$ nên $P(x)=(x-x_1)(x-x_2)...(x-x_n)$.
    Do $P(Q(x))$ vô nghiệm nên $Q(x)\ne x_i\forall x\in \mathbb{R}(i=1,2,...,n)$.
    $\iff x^2+x-2009-x_i\ne 0,\forall x\in \mathbb{R}$.
    $\iff \Delta<0\iff \frac{1}{4}<2009-x_i(i=1,2,...,n)$.
    Vậy $P(2009)=(2009-x_1)(2009-x_2)...(2009-x_n)>\frac{1}{4^n}$.
    Bài 12: Cho đa thức $f(x)$ bậc $2003$ và $f(k)=\frac{k^2}{k+1}$ với $k=1,2,3,...,2004$. Hãy tính $f(2005)$.
    Xét đa thức $g(x)=(x+1)f(x)-x^2$. Khi đó bậc của $g(x)$ là $2004$ và $g(k)=0$ với $k=1,2,...,2004$. Vậy nên $g(x)=a(x-1)(x-2)...(x-2004)$.
    Từ điều kiện $g(-1)=-1$ ta thu được $-1=a.(-2)(-3)....(-2005)$. Hay $a=\frac{-1}{2005!}$.
    Do đó $g(x)=-\frac{(x-1)(x-2)...(x-2004)}{2005!}$.
    Vậy nên: $f(2005)=\frac{g(2005)+2005^2}{2005+1}=\frac{-1+2005^3}{2005.2006}
    Bài 13: Chứng minh rằng mỗi đa thức mônic( là đa thức với hệ số cao nhất bằng $1$) bậc $n$ với các hệ số thực là trung bình cộng của hai đa thức mônic bậc $n$ với $n$ nghiệm thực.
    Gọi $f(x)$ là đa thức mônic hệ số thực bậc $n$. Nếu $n=1$ thì $f(x)=x+a(a\in \mathbb{R})$. Dễ thấy rằng $f(x)$ là trung bình cộng của $f_1(x)=x+2a$ và $f_2(x)=x$, mỗi đa thức có $1$ nghiệm thực. Bài toán được giải với $n=1$.
    Nếu $n>1$ chọn đa thức: $g(x)=(x-2)(x-4)...(x-2(n-1))$.
    Bậc của $g(x)$ là $n-1$. Xét các đa thức: $P(x)=x^n-kg(x)$ và $Q(x)=2f(x)-x^n+kg(x)$.
    Ta sẽ chứng minh rằng với $k$ đủ lớn những đa thức này sẽ có $n$ nghiệm thực.
    Xét các giá trị của đa thức $g(x)$ tại $n$ điểm $x=1,3,5,...,2n-1$. Những giá trị này đan dấu và có giá trị tuyệt đối nhỏ nhất bằng $1$( vì nhiều nhất hai trong các thừa số có giá trị tuyệt đối bằng $1$ và các thừa số khác có giá trị tuyệt đối lớn hơn $2$). Mặt khác, tồn tại hằng số $c>0$ sao cho với $0\le x\le n$, ta có: $[x^{n}]<c$ và $|2f(x)-x^n|<c$. Lấy $k>c$. Thế thì ta thấy rằng $P(x)$ và $Q(x)$ với giá trị đan dấu tại $n$ điểm $x=1,2,3,...,2n-1,...$ cho nên mỗi đa thức $P(x)$ và $Q(x)$ đều có ít nhất $n-1$ nghiệm thực. Chúng là đa thức bậc $n$, nên mỗi đa thức đó phải có $n$ nghiệm thực. Lại vì chúng là mônic và trung bình cộng chúng là $f(x)$ nên ta có điều phải chứng minh.
    Bài 14: Giả sử $f(x)=x^n+5x^{n-1}+3$ trong đó $n$ là một số nguyên lớn hơn $1$. Chứng minh rằng $f(x)$ không viết được dưới dạng tích của hai đa thức có bậc không nhỏ thua $1$ với các hệ số nguyên.
    Bổ đề: Cho đa thức: $f(x)=a_0+a_1x+...+a_nx^n(n>1)$ trong đó $a_0,a_1,...,a_n$ là những số nguyên và một số nguyên tố $p$ thỏa mãn các điều kiện sau:
    1) $a_n$ không chia hết cho $p$.
    2) $a_0,a_1,...,a_k$ chia hết cho $p(0\le k\le n)$
    3) $a_0$ không chia hết cho $p^2$.
    Nếu f(x) viết được dưới dạng tích của hai đa thức với các hệ số nguyên thì bậc của một trong hai đa thức đó không nhỏ hơn $k+1$.
    Chứng minh: Giả sử $f(x)=h(x)g(x)$ trong đó: $h(x)=b_0+b_1x+...+b_mx^m$ và $g(x)=c_0+c_1x+...+c_{n-m}x^{n-m}$.
    Ta có: $a_0=b_0c_0$. Vì $a_0\vdots p$ và không chia hết cho $p^2$ nên trong hai số $b_0,c_0$ có một số chia hết cho $p$ và số còn lại không chia hết cho $p$. Giả sử: $b_0\vdots p$ và $c_0$ không chia hết cho $p$. Vì $a_n=b_m.c_{n-m}$ không chia hết cho $p$ nên $b_m$ cũng không chia hết cho $p$. Gọi $i_0$ là số nhỏ nhất trong số các số $i$ thỏa mãn $b_i$ không chia hết cho $p$ thì $0<i_0\le m$. Ta có: $a_{i_0}=c_0b_{i_0}+\sum\limits_{i+j=i_0;0\le i<i_0}b_ic_j$ không chia hết cho $p$.
    Vậy $i_0\ge k+1$. Mặt khác $m\ge i_0$, vậy $m\ge k+1(dpcm)$ với $k=n-1$ ta nhận được tiêu chuẩn Eisenstein: Cho đa thức $f(x)=a_0+a_1x+...+a_nx^n(n\ge 1)$, trong đó $a_i\in \mathbb{Z}(i=01,2,...,n); a_0,a_1,...,a_{n-1}$ chia hết cho số nguyên tố $p$; $a_0$ không chia hết cho $p^2$ và $a_n$ không chia hết cho $p$. Khi đó $f(x)$ bất khả quy trên $Q$.
    Bổ đề chứng minh hoàn tất.
    Quay lại bài toán: Giả sử $f(x)=f_1(x)f_2(x)$ trong đó $f_1(x),f_2(x)$ là các đa thức với hệ số nguyên và có bậc không nhỏ thua $1$.
    Đa thức $f(x)$ thỏa mãn các giả thiết của bổ đề với $p=2,k=n-2$. Vậy trong các đa thức $f_1(x)$ và $f_2(x)$ có một đa thức có bậc không nhỏ hơn $n-1$. Khi đó $f_2(x)$ có bậc bằng $1$. Vậy $f_2(x)$ có nghiệm nguyên $(f_2(x)=x+b)$ với $b\in \mathbb{Z}$. Giả sử $x=x_0(x_0\in \mathbb{Z})$ là nghiệm nguyên của $f_2(x)$ thì $x_0$ phải là ước số của $3$. Từ đó suy ra $x_0=\pm 1,\pm 3$. Mặt khác dễ dàng chứng minh được, $f_2(1)\ne 0,f_2(-1)\ne 0,f_2(3)\ne 0,f_2(-3)\ne 0$. Điều này vô lí chứng tỏ rằng $f(x)$ không viết được dưới dạng tích của hai đa thức có bậc không nhỏ thua $1$ với các hệ số nguyên.
    Bài 15: Đa thức $P(x)$ bậc $n$ với hệ số thực có $n$ nghiệm thực phân biệt. Hỏi $P(x)$ có thể có nhiều nhất bao nhiêu hệ số bằng $0$?
    Ta gọi $S(n)$ là số hệ số bằng $0$ nhiều nhất mà $1$ đa thức hệ số thực bậc $n$ với $n$ nghiệm thực phân biệt có thể có, $T(P)$, với $P$ là một đa thức là số hệ số bằng $0$ của $P$.
    Ta có một số tính chất sau:
    (i) $S(1)=1$( do $S(1)\le 1$ và xét đa thức $P(x)=x$ có $1$ hệ số bằng $0$)
    (ii) $S(2)=1$( do mọi đa thức $P(x)=ax^2+bx+c(a\ne 0)$ không thể có $b=c=0$ và có $2$ nghiệm phân biệt).
    (iii) Nếu đa thức $P(x)$ bậc $1$ có $T(P)=1$ thì $P(0)=0$.
    (iv) Giả sử $P(x)=\sum\limits_{i=0}^{n}a_ix^i$ thì $P'(x)=\sum\limits_{i=1}^{n}ia_ix^{i-1}$. Nên nếu $T(P)=k$ thì $T(P')=k$ hoặc bằng $k-1$ và $T(P')=k-1\iff P(0)=0$.
    Ta tìm $S(n)$ thông qua quy nạp theo $n$.
    Qua $(i),(ii),(iii)$ ta kiểm chứng được giả thiết quy nạp:
    $\left\{\begin{array}{I} S(2k)=k\\ S(2k-1)=k-1\forall k=\overline{1;N}\\ \text{ Nếu } deg(P)=2k-1 \text{ và } T(P)=k \text{ thì } P(0)=0\end{array}\right.$
    Ta sẽ chứng minh: $\left\{\begin{array}{I} S(2N+1)=N+1(*)\\ S(2N+2)=N+1(**)\\ \text{ Nếu } deg(P)=2N+1 \text{ và } T(P)=N+1 \text{ thì } P(0)=0(***)\end{array}\right.$
    Thật vậy, bắt đầu với $(*)$ và $(***)$. Xét đa thức $P$ thỏa $deg(P)=2N+1;P$ có $2N+1$ nghiệm thực phân biệt. Ta có $P'$ là $1$ đa thức bậc $2N$ có $2N$ nghiệm thực phân biệt (Giữa $2$ nghiệm của $P$ luôn có $1$ nghiệm của $P'$ theo định lý Rolle). Nên ta có: $T(P)\le T(P')+1\le S(2N)+1=N+1$. Để dấu $=$ xảy ra thì theo $(iv): P(0)=0$.
    Ta kiểm chứng được sự tồn tại của dấu bằng qua đẳng thức: $P(x)=x(x^2-1)(x^2-2)...(x^2-N)$. Vậy $S(2N+1)=N+1$.
    Ta tiếp tục với $(**)$. Xét đa thức $P$ thỏa $deg(P)=2N+2$; $P$ có $2N+2$ nghiệm thực phân biệt. Ta có $P'$ là $1$ đa thức bậc $2N+1$ có $2N+1$ nghiệm thực phân biệt.
    Ta có: $T(P)\le T(P')+1\le S(2N)+1=N+2$. Giả sử tồn tại đa thức $P$ như vậy có $T(P)=N+2$. Suy ra:
    $\left\{\begin{array}{I} P(0)=0\\ T(P')=N+1\text{ nên } P'(0)=0\end{array}\right.$. Suy ra $P$ có nghiệm kép là $0$ (loại).
    Vậy suy ra: $T(P)\le N+1$. Ta kiểm chứng được sự tồn tại của dấu bằng qua đa thức:
    $P(x)=(x^2-1)(x^2-2)...(x^2-N)(x^2-N-1)$. Vậy $S(2N+2)=N+1$.
    Vậy ta suy ra: $S(2n+1)=n+1$ và $S(2n+2)=n+1$ với mọi $n\in \mathbb{N}$.
    Hay số hệ số bằng $0$ nhiều nhất mà một đa thức hệ số thực bậc $n$ với $n$ nghiệm phân biệt có thể có là $[\frac{n+1}{2}]$.
     
    Chỉnh sửa cuối: 26/5/19